You are on page 1of 45

JEST 2020

PHYSICS
SOLUTIONS
Scienceteen ®

Education
www.scienceteen.com
SCIENCETEEN EDUCATION

JEST 2020 Physics


Solutions
www.scienceteen.com

Written and compiled by


Prashant Kumar Sinha
Part-A: 1-Mark Questions

1. If 𝑓 𝑡 is a real even function of 𝑡, which one of the following statements is true about its Fourier
transform 𝐹 𝜔 (here ∗ indicates complex conjugation) ?

A. 𝐹∗ 𝜔 = −𝐹 𝜔
B. 𝐹∗ 𝜔 =𝐹 𝜔
C. 𝐹 −𝜔 =𝐹 𝜔
D. 𝐹 −𝜔 = 𝐹∗ 𝜔

Solution:
Fourier Transform
𝑓(t) 𝐹 𝜔

1
𝐹 𝜔 = eiωt 𝑓 𝑡 𝑑𝑡

−∞


1
𝐹 −𝜔 = e−iωt 𝑓 −𝑡 𝑑𝑡

−∞

1
𝐹 −𝜔 = e−iωt 𝑓 𝑡 𝑑𝑡 𝑆𝑖𝑛𝑐𝑒, 𝑓 −𝑡 = 𝑓(𝑡)

−∞


1
𝐹 −𝜔 = e−iωt 𝑓 ∗ 𝑡 𝑑𝑡 𝑆𝑖𝑛𝑐𝑒, 𝑓 ∗ 𝑡 = 𝑓(𝑡)

−∞

𝐹 −𝜔 = 𝐹 ∗ (𝜔)

2. A particle of mass 𝑚 carrying angular momentum 𝑙 moves in a central potential 𝑟 =


𝑘𝑒 −𝑎𝑟
− 𝑟 , where 𝑘, 𝑎 are positive constants. If the particle undergoes circular motion, what is
the equation determining its radius 𝑟𝑜 ?

𝑙2
(A) = 𝑘𝑎𝑟𝑜 𝑒 −𝑎𝑟𝑜
𝑚𝑟𝑜

𝑙2
(B) = 𝑘𝑒 −𝑎𝑟𝑜 (1 + 𝑎𝑟𝑜 )
𝑚𝑟𝑜

𝑙2
(C) = 𝑘𝑒 −𝑎𝑟𝑜
2𝑚𝑟𝑜
𝑙2
(D) = 𝑘𝑒 −𝑎𝑟𝑜 (1 + 𝑎𝑟𝑜 )
2𝑚𝑟𝑜

𝑘𝑒 −𝑎𝑥
Solution: 𝑉(𝑟) = −
𝑟

Particle undergoes circular motion,

𝑚𝑣 2
𝐹= , 𝑙 = 𝑚𝑣𝑟
𝑟

𝜕𝑉 𝑑
𝐹=− = −𝑘𝑟 −1 𝑒 −𝑎𝑟
𝜕𝑟 𝑑𝑥
1 −𝑎𝑟
= −𝑘 − 2
𝑒 + 𝑟 −1 −𝑎 ∙ 𝑒 −𝑎𝑟
𝑟

1 −𝑎𝑟 −1 −𝑎𝑟
𝑚𝑣 2
=𝑘 − 𝑒 + 𝑟 −𝑎 ∙ 𝑒 =
𝑟2 𝑟

⟹ 𝑘𝑒−𝑎𝑟 + 𝑘𝑎𝑟𝑒−𝑎𝑟 = 𝑚𝑣2 𝑟

𝒍𝟐
⟹ 𝑙2 = 𝑚2 𝑣 2 𝑟 2 ⟹ = 𝒎𝒗𝟐 𝒓
𝒎𝒓

𝒍𝟐
⟹ =𝑘 𝑒−𝑎𝑟 + 𝑎𝑟𝑒−𝑎𝑟 = 𝑘(1 + 𝑎𝑟)𝑒−𝑎𝑟
𝒎𝒓

𝒍𝟐
⟹ = 𝑘(1 + 𝑎𝒓𝒐 )𝑒−𝑎𝒓𝒐
𝒎𝒓𝒐

3. A ring of radius 0.5 m has a gap of 0.002𝜋 m. If the ring carries a charge of +1.0 C
distributed uniformly along it, then the electric field at the centre of the ring is

(A) 7.5 × 107 𝑁𝐶 −1


(B) 7.2 × 107 𝑁𝐶 −1
(C) 6.2 × 107 𝑁𝐶 −1
(D) 6.5 × 107 𝑁𝐶 −1

Solution:
𝑄
𝑑𝐸𝑥 = 𝑑𝐸 cos 𝜃 𝑑𝑞 = 2𝜋𝑟 𝑑𝑙
𝑑𝑞
𝑑𝐸𝑥 = 𝑘 𝑟 2 cos 𝜃

𝑘 cos 𝜃 𝑄
⇒ 𝐸𝑥 = 𝑑𝑙
𝑟 2 2𝜋𝑟

𝑘𝑄
= cos 𝜃 𝑑𝑙
2𝜋𝑟 3
𝜃𝑜
𝑘𝑄 2
= 𝑟 cos 𝜃 𝑑𝜃
2𝜋𝑟 3 𝜃
− 𝑜 2

𝑘𝑄 𝜃𝑜 𝑘𝑄 𝜃𝑜
2
= 𝑟 sin 𝜃 −𝜃𝑜 = ∙ 2sin
2𝜋𝑟 3 2 2𝜋𝑟 2 2

𝑘𝑄 0.002𝜋 𝑙 0.002𝜋
= 2
× 𝑙 = 0.002𝜋 , 𝜃𝑜 = =
2𝜋𝑟 𝑟×2 𝑟 𝑟

9 × 109 × 1.0 × 0.001


=
0.53

= 7.2 × 107 𝑁𝐶 −1

4. The 2’s compliment of 1111 1111 is

(A) 0000 0001


(B) 0000 0000
(C) 1111 1111
(D) 1000 0000

Solution:

To find 1’s complement: Change 0 to 1 and 1 to 0

1′ 𝑠 𝑐𝑜𝑚𝑝𝑙𝑒𝑚𝑒𝑛𝑡
1111 1111 0000 0000

To find 2’s complement: 1’s complement + 1

= 0000 0000 + 1 = 0000 0001

5. Consider a system of two particles at temperature 𝑇 → ∞. Each of them can cccupy three
different quantum energy levels having energies 0, 𝜖 and 2𝜖, and both of them cannot
occupy the same energy level. What is the average energy of the system?

(A) 𝜖
(B) 3𝜖 2
(C) 2𝜖
(D) 4𝜖

Solution:

The possible configurations can be represented by the energy level diagram as:

The partition function is given by,

𝑧 = 𝑒 −𝛽𝜖 + 𝑒 −2𝛽𝜖 + 𝑒 −3𝛽𝜖

𝜕
𝐸 =− ln 𝑧
𝜕𝛽

𝜕
=− ln( 𝑒 −𝛽𝜖 + 𝑒 −2𝛽𝜖 + 𝑒 −3𝛽𝜖 )
𝜕𝑥

(−𝜖𝑒 −𝛽𝜖 − 2𝜖𝑒 −2𝛽𝜖 − 3𝜖𝑒 −3𝛽𝜖 )


=−
𝑒 −𝛽𝜖 + 𝑒 −2𝛽𝜖 + 𝑒 −3𝛽𝜖

(𝜖 + 2𝜖 + 3𝜖) 6𝜖
= = = 2𝜖
1+1+1 3

6. Consider three infinitely long, straight, and coplanar wires which are placed parallel to each ottther.
The distance between the adjacent wires is 𝑑. Each wire carries a current 𝐼 in the same direction.
Consider points on either side of the middle wire where the magnetic field vanishes. What is the
distance of these points from the middle wire?

2𝑑
(A) 3
2𝑑
(B) 3
𝑑
(C) 3
𝑑
(D) 3

Solution: Ans (d)

𝜇𝑜 𝐼
𝐵𝐴 = 𝐼𝑛 𝑡𝑕𝑒 𝑝𝑙𝑎𝑛𝑒 𝑜𝑓 𝑡𝑕𝑒 𝑝𝑎𝑔𝑒
2𝜋 𝑑 − 𝑟

𝜇𝑜 𝐼
𝐵𝐵 = 𝑂𝑢𝑡𝑤𝑎𝑟𝑑 𝑡𝑜 𝑡𝑕𝑒 𝑝𝑙𝑎𝑛𝑒 𝑜𝑓 𝑡𝑕𝑒 𝑝𝑎𝑔𝑒
2𝜋𝑟
𝜇𝑜 𝐼
𝐵𝐶 = 𝐼𝑛 𝑡𝑕𝑒 𝑝𝑙𝑎𝑛𝑒 𝑜𝑓 𝑡𝑕𝑒 𝑝𝑎𝑔𝑒
2𝜋 𝑑 + 𝑟

𝜇𝑜 𝐼 1 1 1
⇒𝐵= + − =0
2𝜋 𝑑 + 𝑟 𝑟 𝑑−𝑟

𝑟+𝑑+𝑟 1
⇒ =
𝑟(𝑑 + 𝑟) 𝑑 − 𝑟

⇒ 2𝑟 + 𝑑 𝑑 − 𝑟 = 𝑟(𝑑 + 𝑟)

⇒ 2𝑟𝑑 − 2𝑟 2 + 𝑑 2 − 𝑑𝑟 = 𝑟𝑑 + 𝑟 2

𝑑
⇒ 𝑑 2 = 3𝑟 2 ⇒ 𝑟 =
3

7. Calculate the collector current and determine whether or not the transistor in figure shown
below is in saturation. Assume 𝑉𝐶𝐸 𝑠𝑎𝑡 = 0.2 𝑉.

(A) 6.5 mA, not in saturation


(B) 11.5 mA, in saturation
(C) 11.5 mA, not in saturation
(D) 6.5 mA, in saturation
Solution: Ans (A)

𝑉𝐵𝐵 = 𝐼𝐵 𝑅𝐵 + 𝑉𝐵𝐸

⇒ 2 = 𝐼𝐵 . 10000 + 0.7

2 − 0.7
𝐼𝐵 = 𝐴
10000

= 1.3 × 10−4 𝐴

𝐼𝐶 = 𝛽𝐼𝐵
= 50 × 1.3 × 10−4 𝐴
= 65 𝑚𝐴

Now,

𝑉𝐶𝐶 = 𝐼𝐶 𝑅𝐶 + 𝑉𝐶𝐸

⇒ 10 = 1000 𝐼𝑐 𝑠𝑎𝑡 + 0.2

9.8 𝑉
𝐼𝑐 𝑠𝑎𝑡 = = 9.8 𝑚𝐴
1000Ω

𝐼𝑐 𝑠𝑎𝑡 < 𝐼𝑐

Hence 6.5 mA, Not in Saturation

8. An 𝑛 × 𝑛 Herminitian Maxrix 𝐴 is not a multiple of the identity matrix.Which one of the following
statements is always true?

(A) 𝑛𝑇𝑟 𝐴2 = 𝑇𝑟 𝐴 2

(B) 𝑛𝑇𝑟 𝐴2 < 𝑇𝑟 𝐴 2

(C) 𝑛𝑇𝑟 𝐴2 > 𝑇𝑟 𝐴 2

(D) 𝑇𝑟 𝐴2 = 𝑛 𝑇𝑟 𝐴 2

Solution: Ans (C)

Let A be any arbitrary herimitian matrix.

0 −𝑖 0 0 𝑖 0
𝐴= 𝑖 1 2𝑖 ⇒ 𝐴𝑇 = −𝑖 1 −2𝑖
0 −2𝑖 0 0 2𝑖 0
0 −𝑖 0
∴ 𝐴𝑇 = (𝐴𝑇 )∗ = 𝑖 1 2𝑖
0 −2𝑖 0
0 −𝑖 0 0 −𝑖 0
𝐴2 = 𝐴 ∙ 𝐴 = 𝑖 1 2𝑖 𝑖 1 2𝑖
0 −2𝑖 0 0 −2𝑖 0

1 −𝑖 2
⇒ 𝐴2 = 𝑖 −2 2𝑖 ⇒ Tr 𝐴2 = 1 − 2 + 4 = 3
2 −2𝑖 4

⇒ (Tr𝐴)2 = 0 + 1 + 0 = 1

𝑛 = 3, ∴ 𝑛 Tr 𝐴2 =3×3>1

⇒ 𝑛 Tr 𝐴2 > (Tr𝐴)2

9. The ratio of maximum to minimum resistance that can be obtained with N number of 3-Ω resistors
is
(A) N
(B) N2
(C) N3
(D) N4

Solution: Ans (B)

Maximum resistance will be in case of series combination and minimum resistance will in
the case of parallel connection.

𝑅𝑆𝑒𝑟𝑖𝑒𝑠 = 3 × 𝑁

1 1 1 1 1 𝑁
= + + + ⋯+ =
𝑅// 3 3 3 3 3

3
𝑅// =
𝑁
𝑅𝑆𝑒𝑟𝑖𝑒𝑠 3𝑁
⇒ = = 𝑁2
𝑅// 3
𝑁

10. A free particle of energy E, characterized by a plane wave of wavelength 𝜆 enters a region of
constant potential –V (where E > V> O). Within the region of the potential. The wavelength of the
particle is 𝜆/2. The Ratio V/E is:

(A) -1/3
(B) -3
(C) 3
(D) 1/3

Solution: Ans (B)


2
𝑕
𝑝2 𝜆 𝑕2
𝐸= = =
2𝑚 2𝑚 2𝑚𝜆2

When particle enters the region of potential −𝑉

𝑝′2
𝐸−𝑉 =
2𝑚
2

𝑕
𝜆
𝑝2 2 4𝑕2
⇒𝐸−𝑉 = = = = 4𝐸
2𝑚 2𝑚 2𝑚𝜆2

𝑉
⇒ −𝑉 = 3𝐸 ⇒ = −3
𝐸

11. Ten persons named A, B, C, D, E, F, G, H, I, J have come for an interview. They are being called
one by one to the interview panel at random. What is the probability that C gives interview before A
and A gives before F?

(A) 1/6
(B) 1/10
(C) 1/8
(D) 1/4

Solution: Ans (D)

A, B, C, D, E, F, G, H, I, J are chosen randomly. Therefore,

1
P(C gives interview before A) =
2

1
Also, P(A gives interview before F) =
2
1 1 1
∴ P(C gives interview before A & A gives interview F) = × =
2 2 4
12. A carbon rod of resistance 𝑅𝑐 and a metal rod of resistance 𝑅𝑚 are connected in series. Let
their linear temperature coefficients of resistivity have magnitudes 𝛼𝑐 and 𝛼𝑚 , respectively.
The condition that the net resistance would be independent of temperature is

(A) 𝑅𝑐 = 𝑅𝑚
(B) 𝛼𝑐 = 𝛼𝑚
(C) 𝑅𝑐 𝑅𝑚 = 𝛼𝑚 𝛼𝑐
(D) 𝑅𝑚 𝑅𝑐 = 𝛼𝑚 𝛼𝑐

Solution: Ans (C)

The resistance in series at a reference temperature 𝑇𝑜 is given by 𝑅𝑜 = 𝑅𝑐 + 𝑅𝑚

At a given temperature T the resistance will be



For the metal rod 𝑅𝑚 = 𝑅𝑚 [1 + 𝛼𝑚 𝑇 − 𝑇𝑜 ] and for the carbon the resistance decreases
as the temperature is increased, So,

𝑅𝑐′ = 𝑅𝑐 [1 − 𝛼𝑐 𝑇 − 𝑇𝑜 ]

Total resistance R will be, 𝑅 = 𝑅𝑐′ + 𝑅𝑚


= 𝑅𝑐 1 − 𝛼𝑐 𝑇 − 𝑇𝑜 + 𝑅𝑚 [1 + 𝛼𝑚 𝑇 − 𝑇𝑜 ]

= 𝑅𝑐 + 𝑅𝑚 + −𝑅𝑐 𝛼𝑐 + 𝑅𝑚 𝛼𝑚 𝑇 − 𝑇𝑜

= 𝑅𝑜 + 𝑅𝑚 𝛼𝑚 −𝑅𝑐 𝛼𝑐 𝑇 − 𝑇𝑜

For the net resistance to be independent of temperature,

𝑅𝑚 𝛼𝑚 −𝑅𝑐 𝛼𝑐 = 0
Therefore, 𝑅𝑐 𝑅𝑚 = 𝛼𝑚 𝛼𝑐

13. A particle is to slide along the horizontal circular path on the inner surface of the funnel as shown in
the figure. The surface of the funnel is frictionless. What must be the speed of the particle (in terms
of 𝑟 and ) if it is to execute the motion?

(A) 𝑟𝑔 sin 𝜃
(B) 𝑟𝑔 𝑐𝑜𝑠𝜃
(C) 𝑟𝑔 𝑡𝑎𝑛𝜃
(D) 𝑟𝑔 𝑐𝑜𝑡𝜃
Solution: Ans (D)

𝑁 𝑠𝑖𝑛 𝜃 = 𝑚𝑔

𝑚𝑣 2 𝑔𝑟
𝑁 cos 𝜃 = ⇒ tan 𝜃 = 2
𝑟 𝑣

𝑔𝑟
⇒𝑣= = 𝑔𝑟 cot 𝜃
tan 𝜃

14. Charges are placed as follows: 𝑞 at (𝑎, 𝑎, 0) and (−𝑎, −𝑎, 0), and −𝑞 at (𝑎, −𝑎, 𝑂) and −𝑎, 𝑎, 0 .
At large distances, how does the electrostatic potential behave as a function of the distance 𝑟 from
the centre (0,0,0)?

(A) 1/𝑟 3
(B) 1 𝑟 2
1
(C)
𝑟
(D) 1/𝑟 4

Solution: Ans (A)

The potential of a quadrupole is given by,

1 1 1
𝑉 𝑟 = 𝑄 𝑟𝑟
4𝜋𝜖𝑜 𝑟 3 2 𝑖𝑗 𝑖 𝑗

For Monopole 𝑄𝑖 = 0

For Dipole 𝑄𝑖 𝑟𝑖 = 0

Since, 𝑞 𝑎𝑖 + 𝑎𝑗 + 𝑞 −𝑎𝑖 − 𝑎𝑗 − 𝑞 𝑎𝑖 − 𝑎𝑗 − 𝑞 −𝑎𝑖 + 𝑎𝑗

= 𝑞𝑎 𝑖 + 𝑗 − 𝑖 − 𝑗 − 𝑖 + 𝑗 + 𝑖 − 𝑗 = 0

Therefore potential due to dipole is zero, Let’s check the potential due to quadrupole.

1
𝑄𝑖𝑗 𝑟𝑖 𝑟𝑗 ≠ 0 ⇒ 𝑉 ∝
𝑟3

15. Consider an ideal gas whose entropy is given by


𝑛 𝑈 𝑉
𝑆= 𝜍 + 5𝑅 ln + 2𝑅 ln
2 𝑛 𝑛

Where 𝑛 is the number of moles, 𝜍 is a constant, 𝑅 is the universal gas constant, 𝑈 is the internal
energy and 𝑉 is the volume of the gas. The specific heat at constant pressure is then given by

5
(A) 𝑛𝑅
2

7
(B) 𝑛𝑅
2

3
(C) 𝑛𝑅
2

(D) 𝑛𝑅

Solution: Ans (B)

𝑛 𝑈 𝑉
𝑆= 𝜍 + 5𝑅 ln + 2𝑅 ln
2 𝑛 𝑛

𝑑𝑄 = 𝑑𝑈 + 𝑃𝑑𝑉 ⇒ 𝑑𝑈 = 𝑇𝑑𝑆 − 𝑃𝑑𝑉

𝑑𝑆 𝑑𝑉 𝑑𝑉
⇒1=𝑇 −𝑃 , 𝐵𝑢𝑡 = 0 ∵ 𝑉 = 𝐶𝑜𝑛𝑠𝑡
𝑑𝑈 𝑑𝑈 𝑑𝑈

𝑑𝑆 1
⇒ =
𝑑𝑈 𝑇

𝑑𝑆 𝑛 5𝑅𝑛 1 2𝑅𝑛 𝑑𝑉 1
= +
𝑑𝑈 2 𝑈 𝑛 𝑉 𝑑𝑈 𝑛

𝑛𝑅 5 2 𝑑𝑆 5𝑅𝑛 1 5𝑅𝑛 5𝑛𝑅𝑇


= + ×0 ⇒ = ⇒ = ⇒𝑈=
2 𝑈 𝑉 𝑑𝑈 2𝑈 𝑇 2𝑈 2

5𝑛𝑅 5
𝑑𝑈 = 𝑛𝐶𝑉 𝑑𝑇 ⇒ 𝑑𝑈 = 𝑑𝑇 ⇒ 𝐶𝑉 = 𝑛𝑅
2 2
But,
5 7
𝐶𝑃 − 𝐶𝑉 = 𝑅 ⇒ 𝐶𝑃 = 𝑛𝑅 + 𝑛𝑅 = 𝑛𝑅
2 2

16. If 𝑥 and 𝑦 have the joint probability distribution 𝑓 𝑥, 𝑦 = 3/4 + 𝑥𝑦 for 0 < 𝑥, 𝑦 < 𝑥, 𝑦 < 1, and
𝑓 𝑥, 𝑦 = 0 otherwise. What is the probability that 𝑦 assumes a value greater than 1/2, given that 𝑥
is equal to 1/2

(A) 6/16
(B) 7/16
(C) 8/16
(D) 9/16

Solution: Ans (D)


3
+ 𝑥𝑦 0 < 𝑥, 𝑦<1
𝑓 𝑥, 𝑦 = 4
0 𝐸𝑙𝑠𝑒
1 1
𝑦> , 𝑥=
2 2
1 1
3 1 3 𝑦2
𝑃= + 𝑦 𝑑𝑦 = 𝑦 +
1/2 4 2 4 4 1/2

3 1 3 1 1
= + − × −
4 4 4 2 16
4 3 1 9
= − − =
4 8 16 16
17. A rod consists of two equal sections of length ℓ each with coefficient of thermal conductivity 𝑘1 and
𝑘2 , respectively. One end of the rod is kept at a fixed temperature 𝑇1 and the other end at a
temperature 𝑇2 (𝑇1 > 𝑇2 ). If 𝑘2 = 2𝑘1 then the temperature at the interface is

(A) 𝑇 = 2𝑇1 − 𝑇2 /3
(B) 𝑇 = 2𝑇1 + 𝑇2 /3
(C) 𝑇 = 𝑇1 + 2𝑇2 /3
(D) 𝑇 = 𝑇1 − 2𝑇2 /3

Solution: Ans (C)

𝑑𝑄 𝑘1 𝐴(𝑇1 − 𝑇) 𝑘2 𝐴(𝑇 − 𝑇2 )
= =
𝑑𝑡 𝑙 𝑙

⇒ 𝑘1 𝑇1 − 𝑇 = 𝑘2 (𝑇 − 𝑇2 )

But, 𝑘2 = 2𝑘1 Therefore,


𝑘1 𝑇1 − 𝑇 = 2𝑘1 (𝑇 − 𝑇2 )
⇒ 𝑇1 − 𝑇 = 2𝑇 − 2𝑇2

𝑇1 + 2𝑇2
⇒ 3𝑇 = 2𝑇2 + 𝑇1 ⇒ 𝑇 =
3

18. A particle of mass 𝑚 moves in a one-dimensional potential 𝑉 (𝑥) = 𝐹0 𝑥 , where 𝐹0 is a positive


constant. Given the initial conditions, 𝑥 0 = 𝑥0 > 𝑂 and 𝑥 (O) = O, which one of the following
statements is correct?

(A) The particle undergoes simple harmonic motion about the origin with frequency 𝜔 =
2𝜋 𝐹0 /𝑚𝑥0
1
(B) The angular frequency of oscillations of the particle is 𝜔 = 𝜋 𝐹0 /2𝑚𝑥0
2
(C) The particle begins from rest and is accelerated along the positive 𝑥 axis such that 𝑥 𝑡 =
𝑥0 + 𝐹0 𝑡 2 /2𝑚

(D) The angular frequency of oscillations of the particle is independent of its mass

Solution: Ans (C)

𝑉 𝑥 = 𝐹𝑜 𝑥 , 𝑥 0 = 𝑥𝑜 > 0 and 𝑥(0) = 0

𝐹𝑜 𝑥 𝑥≥0
⇒𝑉 𝑥 =
−𝐹𝑜 𝑥 𝑥 < 0

𝜕𝑉 −𝐹𝑜 𝑥≥0
𝐹=− =
𝜕𝑥 𝐹𝑜 𝑥<0

𝐹𝑜 𝐹𝑜 𝐹𝑜
𝑎= ⇒ 𝑣 = 𝑢 + 𝑎𝑡 = 0 + 𝑡⇒𝑣= 𝑡
𝑚 𝑚 𝑚
1 2 1 𝐹𝑜 2
⇒𝑥=𝑥 0 + 𝑎𝑡 = 𝑥𝑜 + 𝑡
2 2𝑚

19. A 16-bit analog to digit converter works in the range 0 − 1 Volt. The least count of the converter is

(A) 0.30 mV
(B) 15.26 mV
(C) 5.44 nV
(D) 15.26 μV

Solution: Ans (D)

𝐹𝑢𝑙𝑙 𝑠𝑐𝑎𝑙𝑒 𝑟𝑎𝑛𝑔𝑒


𝑅𝑒𝑠𝑜𝑙𝑢𝑡𝑖𝑜𝑛 =
2𝑁𝑢𝑚𝑛𝑒𝑟 𝑜𝑓 𝑏𝑖𝑡𝑠
𝑉𝑚𝑎𝑥 − 𝑉𝑚𝑖𝑛 1 − 0
= = 16 = 15.26 × 10−6 V = 15.26 μV
2𝑁 2

20. A particle in a spherically symmetric potential is known to be in an eigenstate of 𝐿2 and 𝐿z with


eigenvalues 𝑙 𝑙 + 1 ℏ2 and 𝑚ℏ, respectively. What is the value of 𝑙, 𝑚 𝐿2𝑥 𝑙, 𝑚 ?

ℏ2 2
(A) (𝑙 + 𝑙 + 𝑚2 )
2

ℏ2 2
(B) (𝑙 + 𝑙)
3
(C) ℏ2 (𝑙2 + 𝑙 − 𝑚2 )

ℏ2 2
(D) (𝑙 + 𝑙 − 𝑚2 )
2

Solution: Ans (D)

𝐿2 = 𝐿2𝑥 + 𝐿2𝑦 + 𝐿2𝑧

2
⇒ 𝐿 = 𝐿2𝑥 + 𝐿2𝑦 + 𝐿2𝑧

𝑙, 𝑚| 𝐿2 |𝑙, 𝑚 = 𝑙(𝑙 + 1)ℏ2

𝑙, 𝑚 𝐿2𝑧 𝑙, 𝑚 = 𝑚ℏ 𝑙, 𝑚 𝐿𝑧 2 𝑙, 𝑚 = 𝑚2 ℏ2

𝐿2 − 𝐿2𝑧 = 𝐿2𝑥 + 𝐿2𝑦

1 2 1
𝐿2𝑥 = 𝐿2𝑦 ⇒ 𝐿2𝑥 = 𝐿 − 𝐿2𝑧 = 𝑙, 𝑚| 𝐿2 − 𝐿2𝑧 |𝑙, 𝑚
2 2

1 ℏ2
= 𝑙 𝑙 + 1 ℏ2 − 𝑚2 ℏ2 = (𝑙 2 + 𝑙 − 𝑚2 )
2 2

21. The solution of the differential equation 𝑦′′ − 2𝑦′ − 3𝑦 = 𝑒 2𝑡 is given as 𝐶1 𝑒 −𝑡 + 𝐶2 𝑒 2𝑡 +


𝐶3 𝑒 3𝑡 ..The value of the coefficients C1, C2, and C3 are:

(A) C1, C2, and C3 are arbitrary


(B) C1, C3 are arbitrary and 𝐶2 = −1/3
(C) C2, C3 are arbitrary and 𝐶1 = −1/3
(D) C1, C2 are arbitrary and 𝐶3 = −1/3

Solution: Ans (A)

𝑦 ′′ − 2𝑦 ′ − 3𝑦 = 𝑒 2𝑡

𝑦 = 𝐶1 𝑒−𝑡 + 𝐶2 𝑒2𝑡 + 𝐶3 𝑒3𝑡

𝑦 ′ = −𝐶1 𝑒−𝑡 + 2𝐶2 𝑒2𝑡 + 3𝐶3𝑒3𝑡

𝑦 ′′ = 𝐶1 𝑒−𝑡 + 4𝐶2 𝑒2𝑡 + 9𝐶3 𝑒3𝑡

⇒ (𝐶1 𝑒 −𝑡 + 4𝐶2 𝑒 2𝑡 + 9𝐶3 𝑒 3𝑡 ) − 2 −𝐶1 𝑒 −𝑡 + 2𝐶2 𝑒 2𝑡 + 3𝐶3 𝑒 3𝑡

−3 𝐶1 𝑒 −𝑡 + 4𝐶2 𝑒 2𝑡 + 9𝐶3 𝑒 3𝑡 = 𝑒 2𝑡
⇒ 𝐶1 + 2𝐶1 − 3𝐶1 𝑒 −𝑡 + 4𝐶2 − 4𝐶2 − 3𝐶2 𝑒 2𝑡 + 9𝐶3 − 6𝐶3 − 3𝐶3 𝑒 3𝑡 = 𝑒 2𝑡

−1
⇒ 0 ∙ 𝑒 −𝑡 − 3𝐶2 𝑒 2𝑡 + 0 ∙ 𝑒 3𝑡 = 𝑒 2𝑡 ⇒ −3𝐶2 = 1 ⇒ 𝐶2 =
3
−1
Hence, 𝐶1 , 𝐶3 are arbitrary and 𝐶2 =
3

22. Two rails of a railroad track are insulated from each other and from the ground, and are connected
by a millivoltmeter. What is the reading of the millivoltmeter when a train travel at the speed of 90
km/hr down the track? Assume that the vertical component of the earth’s magnetic field is 0.2 gauss
and that the trucks are separated by two meters. Use 1 gauss = 10-4 Tesla = 10-4 V.sec/m2.

(A) 10
(B) 1
(C) 0.2
(D) 180

Solution: Ans (B)

𝐵 = 0.2 × 10−4 Tesla

5
𝑣 = 90 × 𝑚/𝑠 = 25 𝑚/𝑠
18

𝜀 = 𝐵𝑙𝑣 = 0.2 × 10−4 × 2 × 25 V = 4 × 25 × 10−5 = 10−3 𝑉 = 1 𝑚V

23. Two tuning forks A and B are stuck instantaneously to obtain Lissajous figures. The figures go
through a complete cycle in 20 s. Fork A is loaded slightly with wax, so that the cycle period
changes to 10 s. If the frequency of fork B is 256.10 Hz, what is the frequency of fork A after
loading?

(A) 256.00 HZ
(B) 256.05 HZ
(C) 256.15 HZ
(D) 256.20 HZ

Solution: Ans (D)

1
𝜈𝐴 − 𝜈𝐵 =
20
1
𝜈 ′ 𝐴 − 𝜈𝐵 =
10

1
𝜈𝐴 = 256.10 + = 256.10 + 0.05 = 256.15
20

1
𝜈 ′ 𝐴 = 256.10 + = 256.10 + 0.1 = 256.20
10

24. The wave function of an electron in one dimension is given by


0, 𝑓𝑜𝑟 𝑥 < 0
𝜓(𝑥) =
2 3𝑒 −𝑥 (1 − 𝑒 −𝑥 ), 𝑓𝑜𝑟 𝑥 ≥ 0

The ratio between the expected position 𝑥 and the most probable position 𝑥𝑚 is

(A) 0.856
(B) 1.563
(C) 2.784
(D) 3.567

Solution: Ans (D)

𝑥
=?
𝑥𝑚

0
2 3𝑒 −𝑥 1 − 𝑒 −𝑥 𝑥 2 3𝑒 −𝑥 1 − 𝑒 −𝑥 𝑑𝑥
𝑥 = ∞
0
4 × 3𝑒 −2𝑥 1 − 𝑒 −𝑥 2 𝑑𝑥

∞ −2𝑥 ∞
0
𝑒 1 + 𝑒 −𝑥 2 𝑥 𝑑𝑥 0
𝑒 −2𝑥 1 + 𝑒 −2𝑥 − 2𝑒 −2𝑥 𝑥 𝑑𝑥
= ∞ −2𝑥 = ∞ −2𝑥
0
𝑒 (1 + 𝑒 −2𝑥 − 2𝑒 −𝑥 )𝑑𝑥 0
(𝑒 + 𝑒 −4𝑥 − 2𝑒 −3𝑥 )𝑑𝑥


0
𝑥𝑒 −2𝑥 + 𝑥𝑒 −4𝑥 − 2𝑥𝑒 −3𝑥 𝑑𝑥
= ∞ −2𝑥
0
(𝑒 + 𝑒 −4𝑥 − 2𝑒 −3𝑥 )𝑑𝑥


𝑥𝑒 −2𝑥 𝑒 −2𝑥 𝑥𝑒 −4𝑥 𝑒 −4𝑥 2𝑥𝑒 −3𝑥 𝑒 −3𝑥
−2 + −4 + −4 + −16 − −3 − −9 0
= ∞
𝑒 −2𝑥 𝑒 −4𝑥 2𝑒 −3𝑥
−2 + −4 − −3 0
1 1 1 29
− −4 + −16 − −9
= = 144 = 29 ⇒ 𝑥 = 29
1 1 2 1 12 12
− −2 − 4 − 3 12

The probability of finding the particle is given by,


2
2
𝜓 = 2 3𝑒−𝑥 1 − 𝑒−𝑥 = 4 × 3𝑒 −2𝑥 1 + 𝑒 −2𝑥 − 2𝑒 −𝑥

= 12 𝑒 −2𝑥 + 𝑒 −4𝑥 − 2𝑒 −3𝑥

Most probable position 𝑥𝑚 is the position of maximum probability,

𝑑𝜓2
= 12 −2𝑒 −2𝑥 − 4𝑒 −4𝑥 + 6𝑒 −3𝑥 = 0
𝑑𝑥

⇒ 12 × 2 −𝑒−2𝑥 − 2𝑒−4𝑥 + 3𝑒−3𝑥 = 0

−2𝑥
⇒ 𝑒−2𝑥 −1 − 2𝑒 + 3𝑒−𝑥 = 0

Let , 𝑒 −𝑥 = 𝑦
1
⇒ −1 − 2𝑦 2 + 3𝑦 = 0 ⇒ 2𝑦 2 − 3𝑦 + 1 = 0 ⇒ 𝑦 = 1, 𝑦 =
2

1
𝑒 −𝑥 = 1 ⇒ 𝑥 = 0, 𝑒 −𝑥 = ⇒ 𝑒 𝑥 = 2 ⇒ 𝑥 = 𝑙𝑛2 ⇒ 𝑥𝑚 = 𝑙𝑛2
2

𝑥 29 29
⇒ = 12 = = 3.5
𝑥𝑚 𝑙𝑛2 12 × 0.693

25. Consider a classical harmonic oscillator in thermal equilibrium at a temperature 𝑇. If the spring
constant is changed to twice its value isothermally, then the amount of work done on the system is

(A) 𝑘𝐵 𝑇 𝑙𝑛 2
𝑙𝑛2
(B) 𝑘𝐵 𝑇
2

(C) 2𝑘𝐵 𝑇 𝑙𝑛 2
(D) −𝑘𝐵 𝑇 𝑙𝑛 2

Solution: Ans (D)

Work done on the system is negative, and (D) is the only negative option.
Part-B: 3-Mark Questions

1. A classical gas of N particles is kept at a temperature T and is confined to move on a two-


dimensional surface (𝑥𝑦-plane). If an external linear force field is applied along the 𝑥-axis, then the
partition function of the system will be proportional to

(A) TN
(B) T2N
(C) TN/2
(D) T3N/2

Solution: Ans (B)

The energy of a particle is given by,

𝑝2 𝑝𝑥2 𝑝𝑦2
𝐸= = +
2𝑚 2𝑚 2𝑚

If an external linear force field is applied along the 𝑥-axis, Then the work done 𝑊 = 𝐹𝑥

𝑝2 𝑝𝑥2 𝑝𝑦2
𝐸′ = = + + 𝐹𝑥
2𝑚 2𝑚 2𝑚

Partition function is given by,

1
𝑧= 𝑒 −𝛽𝐸 , 𝑊𝑕𝑒𝑟𝑒 𝛽 =
𝑘𝐵 𝑇

For a single particle the partition will be,


2
𝑝 𝑥2 𝑝 𝑦 1
1 − + +𝐹𝑥
2𝑚 2𝑚 𝑘𝑇
𝑧1 = 2 𝑒 𝑑𝑥 𝑑𝑦 𝑑𝑝𝑥 𝑑𝑝𝑦
𝑕

For a system of N identical particles, the partition function is given by 𝑧 = 𝑧1𝑁


𝑁
𝑝2 𝑝2
𝑦
𝑥 + +𝐹𝑥
−𝛽 2𝑚
1 2𝑚
⇒𝑧= 𝑒 𝑑𝑥 𝑑𝑦 𝑑𝑝𝑥 𝑑𝑝𝑦
𝑕2
𝑁 𝑁

𝑥2 𝑁
1 𝜋 𝜋 −𝛽 𝐹𝑥
= 2𝑁 𝑒 𝑑𝑥
𝑕 𝛽 𝛽 𝑥1
2𝑚 2𝑚

𝑁 𝑁
1 2𝜋𝑚 𝑒 −𝛽 𝐹𝑥
= 2𝑁
𝑕 𝛽 −𝛽𝐹𝑥

𝑁
1 𝑁
𝑒 −∞ − 𝑒 0
= 2𝑁 2𝜋𝑚𝑘𝐵 𝑇
𝑕 −𝐹
𝑘𝐵 𝑇

𝑁
𝑁
1 𝑁
0−1 1 𝑁
𝑘𝐵 𝑇
= 2𝜋𝑚𝑘𝐵 𝑇 = 2𝜋𝑚𝑘𝐵 𝑇 ⇒ 𝑧 ∝ 𝑇2𝑁
𝑕2𝑁 −𝐹 𝑕2𝑁 𝐹
𝑘𝐵 𝑇

2. Analyse the common emitter transistor circuit given in the figure. If the current gain (𝛽) increases
by 50%, the relative change in collector current (𝐼 C) is approximately

(A) 5%
(B) 15%
(C) 20%
(D) 25%

Solution: Ans (B)

5000 𝐼𝐶 + 𝐼𝐵 + 4 × 105 𝐼𝐵 + 1000 𝐼𝐶 + 𝐼𝐵 + 𝑉𝐸𝐸 = 𝑉𝐶𝐶

⇒ 𝑉𝐶𝐶 − 0.7 = 6000 𝐼𝐶 + 𝐼𝐵 + 4 × 105 𝐼𝐵

⇒ 𝑉𝐶𝐶 − 0.7 = 6000 𝐼𝐶 + 406000 𝐼𝐵

𝐼𝐶 = 𝛽𝐼𝐵 = 100𝐼𝐵

⇒ 𝑉𝐶𝐶 − 0.7 = 600,000 𝐼𝐵 + 406,000 𝐼𝐵

𝑉
⇒ 𝑉 = 1006,000 𝐼𝐵 ⇒ 𝐼𝐵 =
1006,000

𝑉 𝑉
⇒ 𝐼𝐶 = 100 × 𝐼𝐵 = 100 × =
1006,000 1006,0
𝛽′ = 150

⇒ 𝑉 = 6000 𝐼′𝐶 + 406,000 𝐼′𝐵 = 6000 × 150 𝐼′𝐵 + 406,000 𝐼′𝐵

𝑉
⇒ 𝑉 = 1306,000 𝐼′𝐵 ⇒ 𝐼′𝐵 =
1306,000

𝑉 15 𝑉
⇒ 𝐼′𝐶 = 𝛽𝐼′𝐵 = 150 × =
1306,000 130,600

15 𝑉 𝑉 15 × 10060 − 130600
∴ ∆𝐼𝐶 = 𝐼′𝐶 − 𝐼𝐶 = − =
130,600 1006,0 130,600 × 10,060

20,300
∆𝐼𝐶 𝑉 20,300
130,600 × 10,060
∴ × 100 = × 100 = × 100 = 15.54 %
𝐼𝐶 𝑉 130,600
10,060

3. A particle moving in two dimension satisfies the equations of motion

𝑥 (𝑡) = 𝑥(𝑡) + 𝑦(𝑡),


𝑦(𝑡) = 𝑥(𝑡) − 𝑦(𝑡),

with 𝑥 0 = 0 What is the ratio of 𝑥(∞)/𝑦(∞)?

(A) 1 − 1/ 2
(B) 1 + 1/ 2
(C) 2 − 1
(D) 2 + 1

Solution: Ans (D)

𝐷𝑥 = 𝑥 + 𝑦 𝑥 0 =0

𝐷𝑦 = 𝑥 − 𝑦

𝐷𝑥 + 𝐷𝑦 = 2𝑥 ⇒ 𝐷𝑦 = 2𝑥 − 𝐷𝑥

⇒ 𝐷 2 𝑥 = 𝐷𝑥 + 𝐷𝑦 = 𝐷𝑥 + 2𝑥 − 𝐷𝑥 ⇒ 𝐷 2 𝑥 = 2𝑥 ⇒ 𝐷 2 𝑥 − 2𝑥 = 0 ⇒ 𝐷 2 − 2 𝑥 = 0

𝐷2 − 2 = 0 ⇒ 𝐷 = ± 2

⇒ 𝑥 = 𝑎𝑒 − 2𝑡 + 𝑏𝑒 2𝑡 ⇒ 𝑥 = − 2𝑎𝑒 − 2𝑡 + 2𝑏𝑒 2𝑡

⇒ 0 = − 2𝑎 + 2𝑏 ⇒ 2 𝑏 − 𝑎 = 0 ⇒ 𝑎 = 𝑏
𝑥 = 𝑎(𝑒 − 2𝑡 +𝑒 2𝑡 )

⇒ 𝐷𝑦 = 2𝑥 − 𝐷𝑥

⇒ 𝐷𝑦 = 2 𝑎 𝑒 − 2𝑡 +𝑒 2𝑡 − 𝐷𝑎(𝑒 − 2𝑡 +𝑒 2𝑡 )

⇒ 𝐷𝑦 = 2 𝑎 𝑒 − 2𝑡 +𝑒 2𝑡 − 𝑎 − 2𝑒 − 2𝑡 + 2𝑒 2𝑡

⇒ 𝐷𝑦 = 𝑎 2 + 2 𝑒− 2𝑡 + 2− 2 𝑒 2𝑡

𝑎 2 + 2 𝑒− 2𝑡 𝑎 2− 2 𝑒 2𝑡
⇒𝑦= +
− 2 2

𝑥 (𝑒 − 2𝑡 + 𝑒 2𝑡 ) 2(𝑒 − 2𝑡
+𝑒 2𝑡
)
⇒ = =
𝑦 − 2 + 2 𝑒 − 2𝑡 + 2 − 2 𝑒 2𝑡 − 2 + 2 𝑒− 2𝑡 + 2− 2 𝑒 2𝑡

2(1 + 𝑒 −2 2𝑡 )
=
2 − 2 − 2 + 2 𝑒 −2 2𝑡

𝑥(∞) 2 2 2+ 2 2 2+ 2 2+ 2
⇒ = = × = = =1+ 2
𝑦(∞) 2− 2 2− 2 2+ 2 4−2 2

4. Consider a cube (see figure) of volume 𝑉 containing 𝑁 molecules each of mass 𝑚 with uniform
density 𝑛 = 𝑁/𝑉. Suppose this system is equivalent to a system of 𝑀 non-interacting gases such
that molecules of the 𝑖th gas are 𝑁𝑖 = 𝑛𝑖 𝑉 in number, each with an identical 𝑦-component of
velocity 𝑣 i . What is the pressure 𝑃 on the surface 
∎
𝐴𝐵𝐶𝐷 of area 𝔸?

𝑀 2
(A) 𝑃 = 𝑚 𝑖=1 𝑛𝑖 𝑣𝑖

𝑀 2
𝑖=1 𝑛 𝑖 𝑣𝑖
(B) 𝑃 = 𝑀 𝑛
𝑖=1 𝑖

𝑚 𝑀 2
𝑖=1 𝑛 𝑖 𝑣𝑖
(C) 𝑃 =
2

𝑀 2
(D) 𝑃 = 2𝑚 𝑖=1 𝑛𝑖 𝑣𝑖

Solution: Ans (D)

Let the momentum of the 𝑖th molecule before collision with the walls be 𝑝𝑖 = 𝑚𝑣𝑖
After the collision with the walls it will return in opposite direction. If the collision is
elastic the momentum will be 𝑝𝑖 = −𝑚𝑣𝑖
∴ Change in momentum after collision ∆𝑝 = 𝑚𝑣𝑖 — 𝑚𝑣𝑖 = 2𝑚𝑣𝑖

∴ Force applied by the wall on a molecule will be

∆𝑝 2𝑚𝑣𝑖 2𝑚𝑣𝑖 2
𝐹𝑖 = = 𝑎 =
∆𝑡 𝑣𝑖 𝑎

⇒ Total force acting on the wall is the sum of all the forces applied by all the molecules is
given by,
𝑀

𝐹= 𝐹𝑖 𝑁𝑖
𝑖=1

∴ Pressure acting on the wall will be,

𝑀 𝑀 𝑀 𝑀 𝑀
2
𝐹 𝑖=1 𝐹𝑖 𝑁𝑖 𝑖=1 2𝑚𝑣𝑖 𝑁𝑖 1 2
2𝑚 2
𝑃= = = = 3 2𝑚𝑣𝑖 𝑁𝑖 = 3 𝑣𝑖 𝑛𝑖 𝑉 = 2𝑚 𝑛𝑖 𝑣𝑖 2
𝐴 𝑎2 𝑎3 𝑎 𝑎
𝑖=1 𝑖=1 𝑖=1

5. A continuous He-Ne laser beam (𝜆 = 632.8 𝑛𝑚) is ‘chopped’, using a spinning aperture, into 1 𝜇𝑠
square pulses. The order-of-magnitude estimate of the spectral width ∆𝜆 of the emerging ‘pulsed’
light is

(A) 10-9 m
(B) 10-12 m
(C) 10-15 m
(D) 10-18 m

Solution: Ans (C)

Coherence length is the propagation distance over which a coherent wave (e.g. an electromagnetic
wave) maintains a specified degree of coherence.
𝑚
The coherence length of coherent pulse is, 𝑙𝑡 = 𝑐𝜏𝑜 = 3 × 108 × 1 × 10−6 𝑆𝑒𝑐 = 300 𝑚
𝑠

𝜆 = 632.8 𝑛𝑚 = 632.8 × 10−9 𝑚

In optical communications, assuming that the source has a Gaussian emission spectrum, the
coherence length L is given by,

1 2 𝑙𝑛2 𝜆2
𝐿= 𝐿𝐹𝑊𝐻𝑀 =
2 𝜋 𝑛Δ𝜆

Where 𝜆 is the central wavelength of the source, 𝑛 is the refractive index of the medium, and Δ𝜆 is
the (FWHM) spectral width of the source.

𝜆2 𝜆2 (632.8 × 10−9 𝑚)2


⇒ 𝐿~ ⇒ Δ𝜆~ ~ = 1.33 × 10−15 𝑚 ⇒ Δ𝜆~10−15 𝑚
𝑛Δ𝜆 𝑛𝐿 1 × 300 𝑚

6. Consider a quantum particle of mass 𝑚 moving in a potential


1
𝑚𝜔2 𝑥 2 + 𝑦 2 , for 𝑥 > 0, 𝑦 > 0
𝑉 𝑥, 𝑦 = 2
∞, Otherwise
.
What is the degeneracy of the energy state 9ℏ𝜔, where 𝜔 > 0 measures the strength of the
potential?

(A) 4
(B) 2
(C) 10
(D) 5

Solution: Ans (A)

1
The energy of a 1D harmonic oscillator is given by, 𝐸 = 𝑛 + 2 ℏ𝜔, 𝑛 = 0, 1, 2, 3 …

1
But for a Half-1D-harmonic oscillator i.e. 𝑥 > 0, 𝐸 = 𝑛 + ℏ𝜔, 𝑛 = 1, 3, 5, 7 …
2

The above problem is the case of 2D-Half harmonic oscillator corresponding to the first
quadrant. So the energy will be given by,

1 1
𝐸 = 𝑛𝑥 + ℏ𝜔 + 𝑛𝑦 + ℏ𝜔 = (𝑛𝑥 + 𝑛𝑦 + 1) ℏ𝜔
2 2

Where, 𝑛𝑥 = 1, 3, 5, 7 … and 𝑛𝑦 = 1, 3, 5, 7 …

When 𝐸 = 9ℏ𝜔 ⇒ 9ℏ𝜔 = 𝑛𝑥 + 𝑛𝑦 + 1 ℏ𝜔

⇒ 𝑛𝑥 + 𝑛𝑦 = 8
⇒ (𝑛𝑥 , 𝑛𝑦 ) ≡ 1, 7 , 3, 5 , 5, 3 , 7, 1

Hence, there are 4 degenerate states having energy 9ℏ𝜔.


7. What is the charge stored on each capacitor C 1 and C2 in the circuit shown in the given figure?

(A) 6𝜇𝐶, 6𝜇𝐶


(B) 6𝜇𝐶, 3𝜇𝐶
(C) 3𝜇𝐶, 6𝜇𝐶
(D) 3𝜇𝐶, 3𝜇𝐶

Solution: Ans (A)

When the capacitors are fully charged, no current flows through the capacitors so they
behave like open circuits.

So, the current in the circuit will be,

12
𝑖= = 1𝐴
6+3+3

Voltage in parallel will be equal and charges in


series will be same in series. So,

𝑉𝐴𝐵 = 𝑉𝐶𝐷 = 𝑉𝐸𝐹


𝑞 𝑞
⇒ 6Ω × 1𝐴 + 3Ω × 1𝐴 =+
2𝜇𝐹 1𝜇𝐹
3𝑞 9×2
⇒9𝑉 = ⇒𝑞= = 6𝜇𝐶
2𝜇𝐹 3

8. A particle of mass 𝑚 is placed in a potential well 𝑈 𝑥 = 𝑐𝑥 𝑛, where 𝑐 is a positive constant and 𝑛


is an even positive integer. If the particle is in equilibrium at a constant temperature, which one of
the following relations between average kinetic energy 𝐾 and average potential energy 𝑈 is
correct?
2
(A) 𝐾 = 𝑈
𝑛
(B) 𝐾 = 𝑈
𝑛
(C) 𝐾 = 𝑈
2
(D) 𝐾 = 2 𝑈
Solution: Ans (C)

If the force between any two particles of the system results from a potential energy U(r) = αrn that is
proportional to some power n of the inter-particle distance r, the virial theorem takes the simple
form:
2𝐾 =𝑛𝑈
Where, 𝐾 = Total Kinetic Energy, 𝑈 = Total potential energy and
𝑈 𝑟 = Potential energy between two particles.
𝑛
∴ 𝐾 = 𝑈
2

9. The Hamiltonian for a particle of mass 𝑚 is given by 𝐻 = (𝑝 − 𝛼𝑞)2 / (2𝑚), where 𝛼 is a non-
zero constant. Which one of the following equations is correct?

(A) 𝑝 = 𝑚𝑞
(B) 𝛼𝑝 = 𝑞
(C) 𝑞 = 0
1
(D) 𝐿 = 𝑚𝑞 2 − 𝛼𝑞𝑞
2

Solution: Ans (C)

𝜕𝐻 𝜕𝐻
𝑝𝑖 = − & 𝑞𝑖 =
𝜕𝑞𝑖 𝜕𝑝𝑖

1 1
⇒𝑞= × 2 𝑝 − 𝛼𝑞 × 1 = 𝑝 − 𝛼𝑞 ⇒ 𝑚𝑞 = 𝑝 − 𝛼𝑞
2𝑚 𝑚
⇒ (A) option is wrong.

1 𝛼 𝛼2
⇒𝑝=− × 2 𝑝 − 𝛼𝑞 −𝛼 = 𝑝 − 𝛼𝑞 ⇒ 𝛼𝑝 = 𝑝 − 𝛼𝑞
2𝑚 𝑚 𝑚
⇒ (B) option is wrong.

1 1 𝛼 𝛼
⇒𝑞= 𝑝 − 𝛼𝑞 = 𝑝 − 𝛼𝑞 − 𝑝 − 𝛼𝑞 = 0
𝑚 𝑚 𝑚 𝑚

⇒ (C) option is correct.

10. The wavefunction of a particle subjected to a spherically symmetric potential 𝑉 (𝑟) is given by
𝜓 𝑟 = 𝑥 − 𝑦 + 2𝑧 𝑓 𝑟 . Which one of the following statements is true about 𝜓 𝑟 ?

(A) It is an eigenfunction of 𝐿2 with 𝑙 = 0


(B) It is an eigenfunction of 𝐿2 with 𝑙 = 1

(C) It is an eigenfunction of 𝐿2 with 𝑙 = 2

(D) It is an not eigenfunction of 𝐿2

Solution: Ans (B)

𝜓 𝑟 = 𝑥 − 𝑦 + 2𝑧 𝑓 𝑟

In spherical coordinates,

𝑥 = 𝑟 sin 𝜃 cos 𝜙 , 𝑦 = 𝑟 sin 𝜃 sin 𝜙 , 𝑧 = 𝑟 cos 𝜃

𝜓 𝑟 = 𝑟 sin 𝜃 cos 𝜙 − 𝑟 sin 𝜃 sin 𝜙 + 2𝑟 cos 𝜃 𝑓 𝑟

= 𝑟 sin 𝜃 cos 𝜙 − sin 𝜃 sin 𝜙 + 2 cos 𝜃 𝑓 𝑟

𝑒 𝑖𝜙 + 𝑒 −𝑖𝜙 𝑒 𝑖𝜙 − 𝑒 −𝑖𝜙
cos 𝜙 = , sin 𝜙 =
2 2𝑖

𝑒 𝑖𝜙 + 𝑒 −𝑖𝜙 𝑒 𝑖𝜙 − 𝑒 −𝑖𝜙
∴ 𝜓 𝑟 = 𝑟𝑓 𝑟 sin 𝜃 − sin 𝜃 + 2 cos 𝜃
2 2𝑖

𝑒𝑖𝜙 + 𝑒−𝑖𝜙 𝑒𝑖𝜙 − 𝑒−𝑖𝜙


⇒ 𝜓 𝑟 = 𝑟𝑓 𝑟 sin 𝜃 − sin 𝜃 + 2 cos 𝜃
2 2𝑖

𝑒𝑖𝜙 𝑒−𝑖𝜙
= 𝑟𝑓 𝑟 sin 𝜃 1 + 𝑖 + sin 𝜃 (1 − 𝑖) + 2 cos 𝜃
2 2

𝑒𝑖𝜙 1 𝑖 𝑒−𝑖𝜙 1 𝑖
= 𝑟𝑓 𝑟 + sin 𝜃 + − sin 𝜃 + 2 cos 𝜃
2 2 2 2 2 2

𝑒𝑖𝜙 𝜋 𝜋 𝑒−𝑖𝜙 𝜋 𝜋
= 𝑟𝑓 𝑟 cos + 𝑖 sin sin 𝜃 + cos − 𝑖 sin sin 𝜃 + 2 cos 𝜃
2 4 4 2 4 4

𝑒𝑖𝜙 𝑖𝜋 𝑒−𝑖𝜙 −𝑖𝜋


= 𝑟𝑓 𝑟 𝑒 4 sin 𝜃 + 𝑒 4 sin 𝜃 + 2 cos 𝜃
2 2
1 𝑖𝜋 −𝑖𝜋
= 𝑟𝑓 𝑟 𝑒 4 sin 𝜃 𝑒𝑖𝜙 + 𝑒 4 sin 𝜃 𝑒−𝑖𝜙 + 2 2 cos 𝜃
2
Now, the normalized angular wave function also called spherical harmonics are given by:

2𝑙+1 (𝑙− 𝑚 )! 𝑖𝑚𝜙 𝑚


𝑌𝑙𝑚 𝜃, 𝜙 = 𝜖 𝑒 𝑃𝑙 (cos 𝜃) where, 𝜖 = (−1)𝑚 for 𝑚 ≥ 0 and 𝜖 = 1 for 𝑚 ≤ 0
4𝜋 (𝑙+ 𝑚 )!
3 1/2
𝑌10 = cos 𝜃 ,
4𝜋
1/2
3
𝑌1±1 = ∓ sin 𝜃 𝑒±𝑖𝜙
8𝜋

1 𝑖𝜋 −𝑖𝜋
∴ 𝜓 𝑟 = 𝑟𝑓 𝑟 𝑒 4 sin 𝜃 𝑒 𝑖𝜙 +𝑒 4 sin 𝜃 𝑒 −𝑖𝜙 + 2 2 cos 𝜃
2

1 𝑖𝜋 8𝜋 −𝑖𝜋 8𝜋 4𝜋
= 𝑟𝑓 𝑟 𝑒 4 − 𝑌11 (𝜃, 𝜙) +𝑒 4 𝑌−1
1 (𝜃, 𝜙) + 2 2 𝑌01 (𝜃, 𝜙)
2 3 3 3

𝑟𝑓 𝑟 8𝜋 𝑖𝜋 −𝑖𝜋
= 𝑒 4 −𝑌11 + 𝑒 4 𝑌−1
1 + 2 𝑌01
2 3

4𝜋 𝑖𝜋 −𝑖𝜋
= 𝑟𝑓 𝑟 −𝑒 4 𝑌11 + 𝑒 4 𝑌−1 0
1 + 2𝑌1
3

Normalization constant for the angular function can be calculated as,

−1 2 + 12 + 22 = 6

𝑖𝜋 −𝑖𝜋
4𝜋 −𝑒 4 𝑒 4 2
∴ 𝜓 𝑟 = 𝑟𝑓 𝑟 6 𝑌11 + 𝑌1−1 + 𝑌01
3 6 6 6

𝑖𝜋 −𝑖𝜋
−𝑒 4 𝑒 4 2
=𝑅 𝑟 𝑌11 + 𝑌1−1 + 𝑌10
6 6 6

Hence, the function of angle is normalized with 𝑙 = 1 (the subscript of 𝑌𝑙𝑚 ). Now, the
eigenfunction of 𝐿2 is given by,

𝐿2 𝑓𝑚
𝑙
𝜃, 𝜙 = ℏ2 𝑙(𝑙 + 1)𝑓𝑚 𝑙
𝜃, 𝜙
= ℏ 1 1 + 1 𝑓𝑙 𝜃, 𝜙 = 2ℏ2 𝑓𝑚
2 𝑚
𝑙
So, the eigenvalue of 𝐿2 is 2ℏ2

11. Three polarizers are stacked, normal to a central axis, along which is incident a beam of unpolarized
light of intensity 𝐼 0. The first and the third polarizers are perpendicular to each other and the middle
polarizer is rotated at an angular frequency 𝜔 about the central axis (light beam). The time-
dependent intensity of light emerging after the third polarized will be given by.

(A) 𝐼 𝑡 = (𝐼0 /16) 1 − cos⁡


(4𝜔𝑡)
(B) 𝐼 𝑡 = (𝐼0 /8) 1 − cos⁡
(2𝜔𝑡)
(C) 𝐼 𝑡 = (𝐼0 /4) 1 − 2cos 𝜔𝑡
(D) 𝐼 𝑡 = (𝐼0 /2) 𝑐𝑜𝑠 2 𝜔𝑡

Solution: Ans (A)

The intensity of light after passing through the first polarizer is

𝐼𝑜
𝐼1 =
2

After passing through the 2nd polarizer the intensity will be,

𝐼2 = 𝐼1 cos 2 𝜃

After passing through the 3rd polarizer the intensity will be,

𝐼1 2 𝐼1 1 − cos 4𝜃
𝐼3 = 𝐼2 cos 2 (90 − 𝜃) = 𝐼2 sin2 𝜃 = 𝐼1 cos 2 𝜃 sin2 𝜃 = sin 2𝜃 =
4 4 2

𝐼𝑜 1 − cos 4𝜃 𝐼𝑜
= = 1 − cos 4𝜔𝑡
8 2 16

12. Consider a particle with total energy E is oscillating in a potential 𝑈(𝑥) = 𝐴 𝑥 n with A > 0 and n > 0
in one dimension. Which one of the following gives the relation between the time period of
oscillation T and the total energy 𝐸:

(A) 𝑇 ∝ 𝐸1/𝑛−1/2
(B) 𝑇 ∝ 𝐸0
(C) 𝑇 ∝ 𝐸𝑛
(D) 𝑇 ∝ 𝐸1/𝑛

Solution: Ans (A)

𝜕𝑈 𝜕2 𝑈
𝑈 ∝ 𝑥𝑛 ⇒ ∝ 𝑥 𝑛 −1 ⇒ ∝ 𝑥 𝑛−2
𝜕𝑥 𝜕𝑥2

𝜕𝑈 𝜕2𝑈
𝐹=− & =𝑘
𝜕𝑥 𝜕𝑥 2
1 1 1
⇒ 𝐹 = −𝑘𝑥 = −𝜔2 𝑥 ⇒ 𝑇 ∝ ⇒𝑇∝ ⇒𝑇∝ 𝑛−2 2
𝑘 𝑥𝑛−2 𝑥
𝑛−2 2
1 1
⇒𝑥 ∝ ⇒ 𝑥 ∝ 2 𝑛−2 ⇒ 𝑥 ∝ 𝑇 2 2−𝑛
𝑇 𝑇

But, Given, 𝐸 ∝ 𝑥 𝑛

𝑛 1 1
⇒ 𝐸 ∝ 𝑇2 2−𝑛
⇒ 𝐸 ∝ 𝑇 2𝑛 2−𝑛
⇒𝑇∝𝐸 2−𝑛 2𝑛
⇒ 𝑇 ∝ 𝐸𝑛 −2

13. What is the value of the following contour integral 𝐼 taken counterclockwise around the circle
𝑧 = 2?
𝑑𝑧
𝐼=
𝑧 3 (𝑧+4)
𝐶

𝜋𝑖
(A) 2
𝜋𝑖
(B) 32
𝜋𝑖
(C) 16
𝜋𝑖
(D) 4

Solution: Ans (B)

𝑧 3 𝑧 + 4 = 0 Gives 𝑧 = 0, 𝑧 = −4 singular points

While 𝑧 = −4 doesn’t lie in the circle so, we will take only 𝑧 = 0

If 𝑓(𝑧) has a pole of order n at 𝑧 = 𝑎, then,

𝟏 𝒅𝒏−𝟏
𝐑𝐞𝐬 𝐟 𝐚 = 𝒛 − 𝒂 𝒏 𝒇(𝒛)
𝒏 − 𝟏 ! 𝒅𝒛𝒏−𝟏

Here, 𝑧 = 0 is a third order pole. So residue at 𝑧 = 0 is,

1 𝑑2 3
1 1 𝑑2 1 1 2 1
𝑧−0 = = =
2! 𝑑𝑧 2 𝑧3 (𝑧 + 4) 𝑧=0
2 𝑑𝑧 2 (𝑧 + 4) 𝑧=0
2 (𝑧 + 4)3 𝑧=0
64

Now, 𝐈 = 𝟐𝛑𝐢(𝐒𝐮𝐦 𝐨𝐟 𝐑𝐞𝐬𝐢𝐝𝐮𝐞𝐬), Therefore,

1 𝜋𝑖
𝐼 = 2𝜋𝑖 × =
64 32
14. The Hamiltonian of a classical particle is given by 𝛨(𝑝, 𝑞) = 𝑝2 /2𝑚 + 𝑘𝑞 2 /2. Given 𝐹 𝑝, 𝑞, 𝑡 =
ln 𝑝 + 𝑖𝑚𝜔𝑞 − 𝑖𝛼𝜔𝑡 is a constant of motion (where 𝜔 = 𝑘/𝑚), what is the value of 𝛼 ?

(A) 2𝜋
(B) 0
(C) 1
(D) 𝜋

Solution: Ans (C)

𝑝2 𝑘𝑞 2
𝛨(𝑝, 𝑞) = +
2𝑚 2

𝜕𝐻 𝜕𝐻 𝑝
𝑝=− = −𝑘𝑞, 𝑞= =
𝜕𝑞 𝜕𝑝 𝑚

𝐹 𝑝, 𝑞, 𝑡 = 𝑙𝑛 𝑝 + 𝑖𝑚𝜔𝑞 − 𝑖𝛼𝜔𝑡

𝜕𝐹 1
= × 𝑝 + 𝑖𝑚𝜔𝑞 − 𝑖𝛼𝜔 = 0
𝜕𝑡 𝑝 + 𝑖𝑚𝜔𝑞
𝑝
𝑝 + 𝑖𝑚𝜔𝑞 −𝑘𝑞 + 𝑖𝑚𝜔 𝑚 −𝑘𝑞 + 𝑖𝜔𝑝
⇒ − 𝑖𝛼𝜔 = 0 ⇒ = 𝑖𝛼𝜔 ⇒ = 𝑖𝛼𝜔
𝑝 + 𝑖𝑚𝜔𝑞 𝑝 + 𝑖𝑚𝜔𝑞 𝑝 + 𝑖𝑚𝜔𝑞

⇒ −𝑘𝑞 + 𝑖𝜔𝑝 = 𝑖𝛼𝜔(𝑝 + 𝑖𝑚𝜔𝑞)

⇒ −𝑘𝑞 + 𝑖𝜔𝑝 = 𝑖𝛼𝜔𝑝 − 𝑚𝜔2 𝛼𝑞)

Equating real and imaginary parts seperately, we get,

𝑘
⇒ 𝑘𝑞 = 𝑚𝜔2 𝛼𝑞 & 𝜔𝑝 = 𝛼𝜔𝑝 ⇒ 𝛼 = & 𝛼=1
𝑚𝜔 2

15. An electromagnetic field is given by


1 𝑞
𝐸 𝑟, 𝑡 = − 𝜃(𝑣𝑡 − 𝑟)𝑟, 𝐵(𝑟, 𝑡) = 0
4𝜋 𝜖𝑜 𝑟 2

1 𝑓𝑜𝑟 𝑥 > 0
Where θ 𝑥 =
0 𝑓𝑜𝑟 𝑥 ≤ 0

The corresponding charge density 𝜌 and current density 𝐽 are given by


𝑞
(A) 𝜌 = −𝑞𝛿 3 𝑟 𝜃 𝑣𝑡 − 𝑟 + 𝜃 𝑣𝑡 − 𝑟 ; 𝑗 = 0
4𝜋𝑟 2
(B) 𝜌 = −𝑞𝛿 3 𝑟 𝜃 𝑣𝑡 − 𝑟 ; 𝑗 = 0
𝑞 𝑞𝑣
(C) 𝜌 = 𝛿 𝑣𝑡 − 𝑟 ; 𝑗 = 𝛿(𝑣𝑡 − 𝑟)𝑟
4𝜋 𝑟 2 4𝜋 𝑟 2

𝑞 𝑞𝑣
(D) 𝜌 = −𝑞𝛿 3 (𝑟)𝜃 𝑣𝑡 − 𝑟 + 𝛿 𝑣𝑡 − 𝑟 ; 𝑗 = 𝛿(𝑣𝑡 − 𝑟)𝑟
4𝜋𝑟 2 4𝜋 𝑟 2

Solution: Ans (D)

𝜌
∇∙𝐸 =
𝜖𝑜

1 𝑞 𝜌
∇∙ − 𝜃(𝑣𝑡 − 𝑟)𝑟 =
4𝜋𝜖𝑜 𝑟2 𝜖𝑜

𝑞 1 𝜌
− ∇ ∙ 2 𝜃(𝑣𝑡 − 𝑟)𝑟 =
4𝜋 𝜖𝑜 𝑟 𝜖𝑜

𝑞 𝑟
⇒𝜌=− ∇ ∙ 𝜃 𝑣𝑡 − 𝑟
4𝜋 𝑟2

𝑞 𝑟 𝑟
⇒𝜌=− 𝜃 𝑣𝑡 − 𝑟 ∇∙ 2 + 2 ∇ ∙ 𝜃 𝑣𝑡 − 𝑟
4𝜋 𝑟 𝑟

𝑟
∇∙ = 4𝜋𝛿3 𝑟
𝑟2

The function given below is Heaviside step function and its derivative is the Dirac delta function,

1 𝑓𝑜𝑟 𝑥 > 0
θ 𝑥 =
0 𝑓𝑜𝑟 𝑥 ≤ 0

𝑑θ 𝑥
= 𝛿(𝑥)
𝑑𝑥

Where 𝛿(𝑥) is the Dirac Delta function given by,

∞ 𝑓𝑜𝑟 𝑥 = 0
𝛿(𝑥) =
0 𝑓𝑜𝑟 𝑥 ≠ 0

𝑞 𝑟
⇒𝜌=− 𝜃 𝑣𝑡 − 𝑟 4𝜋𝛿 3 𝑟 + 2 𝛿 𝑣𝑡 − 𝑟 −𝑟
4𝜋 𝑟

Where,
∇ ∙ 𝜃 𝑣𝑡 − 𝑟 = 𝛿 𝑣𝑡 − 𝑟 ∇ 𝑣𝑡 − 𝑟 = 𝛿 𝑣𝑡 − 𝑟 0 − 𝛻𝑟 = 𝛿 𝑣𝑡 − 𝑟 𝑟

Since,
𝜕 𝜕 𝜕
𝛻𝑟 = 𝑖+ 𝑗+ 𝑘 𝑥2 + 𝑦2 + 𝑧2
𝜕𝑥 𝜕𝑦 𝜕𝑧

1 𝑥𝑖 + 𝑦𝑗 + 𝑧𝑘
= 2 𝑥𝑖 + 𝑦𝑗 + 𝑧𝑘 = =𝑟
2 𝑥2 + 𝑦2 + 𝑧2 𝑥2 + 𝑦2 + 𝑧2

Therefore,

𝑞 1
𝜌=− 4𝜋𝛿3 𝑟 𝜃 𝑣𝑡 − 𝑟 − 𝛿 𝑣𝑡 − 𝑟
4𝜋 𝑟2

𝑞
= −𝑞𝛿3 𝑟 𝜃 𝑣𝑡 − 𝑟 + 𝛿 𝑣𝑡 − 𝑟
4𝜋𝑟2

Now, from Maxwell equation,

1 𝜕𝐸
∇×𝐵 = + 𝜇𝑜 𝐽
𝑐 2 𝜕𝑡

1 1 𝜕𝐸 1 1 𝜕 1 𝑞
⇒𝐽= ∇×𝐵− 2 = ∇×0− 2 − 𝜃 𝑣𝑡 − 𝑟 𝑟
𝜇𝑜 𝑐 𝜕𝑡 𝜇𝑜 𝑐 𝜕𝑡 4𝜋𝜖𝑜 𝑟 2

1 1 𝜕 1 𝑞
= − 2 − 𝜃 𝑣𝑡 − 𝑟 𝑟
𝜇𝑜 𝑐 𝜕𝑡 4𝜋 𝜖𝑜 𝑟 2

𝑞 1 𝜕 1 𝑞 𝜕 1 𝑞 𝜕
= 𝜃 𝑣𝑡 − 𝑟 𝑟 = 𝜃 𝑣𝑡 − 𝑟 𝑟 = 𝜃 𝑣𝑡 − 𝑟 𝑟
4𝜋 𝜖𝑜 𝜇𝑜 𝑐 2 𝜕𝑡 𝑟2 4𝜋 𝜕𝑡 𝑟2 2
4𝜋𝑟 𝜕𝑡
𝑞 𝑞𝑣
= 𝛿 𝑣𝑡 − 𝑟 𝑣𝑟 ⇒ 𝐽 = 𝛿 𝑣𝑡 − 𝑟 𝑟
4𝜋𝑟 2 4𝜋𝑟 2
Part-C: 3-Mark Numerical Questions

1. What is the value of the following integral?

𝜋 /2
100 2
𝐼= 𝑥𝛿(2 𝑠𝑖𝑛𝑥 − 2) 𝑑𝑥
𝜋 0

Solution: Ans (25)

2 𝑠𝑖𝑛𝑥 − 2 = 0

2 𝜋
𝑠𝑖𝑛𝑥 = ⇒𝑥=
2 4
𝜋
𝜋 ∞ 𝑥=
⇒ 𝛿 2 𝑠𝑖𝑛𝑥 − 2 = 𝛿 𝑥 − = 4
4 𝜋
0 𝑥≠
4

1
𝛿 𝑔(𝑥) 𝑑𝑥 = 𝛿 𝑥 − 𝑥𝑜
𝑔′ (𝑥𝑜 )

𝜋 /2
100 2 1 𝜋
𝐼= 𝑥 𝜋 𝛿 𝑥 − 4 𝑑𝑥
𝜋 0 2 cos
4

𝜋 /2
100 2 1 𝜋
= 𝑥𝛿 𝑥 − 𝑑𝑥
𝜋 2 cos 𝜋 0 4
4
𝜋 /2
100 𝜋 100 𝜋
= 𝑥𝛿 𝑥 − 𝑑𝑥 = × = 25
𝜋 0 4 𝜋 4

2. A laser has output power of 150mW with beam diameter of 2 mm at a wavelength 630 nm. What is the
value of the electric field in units of V/m is? Use Coulomb’s constant, 1/ 4𝜋𝜖0 = 9 × 109 N𝑚2 𝐶 −2

Solution: Ans (6000)

Energy density associated with the electric field of an electromagnetic wave is given by,
𝐸𝑛𝑒𝑟𝑔𝑦 1
= 𝜖 𝐸2
𝑉𝑜𝑙𝑢𝑚𝑒 2 𝑜

Volume sweep by light in 1sec = 𝜋𝑟 2 × 𝑐

150 × 10−3 𝐽 1
⇒ −3 2 = 𝜖𝑜 𝐸 2
𝜋 × (1 × 10 ) × 𝑐 2

2 150 × 10−3 𝐽
⇒ 𝐸2 = ×
𝜖𝑜 𝜋 × 1 × 10−3 2 × 𝑐

8 150 × 10−3 𝐽 1200 × 10−3 𝐽


𝐸2 = × = 9 × 109×
4𝜋𝜖𝑜 1 × 10−3 2 × 𝑐 1 × 10−3 2 × 𝑐

1200
= 9 × 109 × = 3 × 104 × 1200 = 36 × 106
10−3 × 3 × 108

⇒ 𝐸 = 6000 𝑉/𝑚

3. A cleaning machine presses a circular mop of radius 𝑅 = 30 𝑐𝑚 vertically down on a floor with a total F
= 25 N and rotates it with a constant angular speed about the vertical axis passing through the centre of
the mop. If the force is distributed uniformly over the mop and if the coefficient of friction between the
mop and the floor is 𝜇 = 0.25, what is the value of the torque in N-cm applied by the machine on the
mop?

Solution: Ans (125)

Force acting per unit area is given by,

𝐹 𝐹
= = 𝜍, 𝑓𝑟 = 𝜇𝐹
𝐴 𝜋𝑅 2

The torque acting on the infinitesimally small area 𝑑𝐴 is given by,

𝑑𝜏 = 𝑟 × 𝐹 = 𝑟 ∙ 𝜇𝜍 ∙ 𝑑𝐴

⇒ 𝑑𝜏 = 𝑟 ∙ 𝜇𝜍 ∙ 𝑟𝑑𝜃 ∙ 𝑑𝑟 = 𝜇𝜍𝑟 2 𝑑𝜃
𝑅 2𝜋 𝑅 2𝜋
⇒𝜏= 𝜇𝜍𝑟 2 𝑑𝑟𝑑𝜃 = 𝜇𝜍 𝑟 2 𝑑𝑟𝑑𝜃
0 0 0 0

𝜇𝐹 𝑅 3 2 2
= 2 × × 2𝜋 = 𝜇𝐹𝑅 = × 0.25 × 25 × 30 = 125 𝑁𝑐𝑚
𝜋𝑅 3 3 3
4. Analyze the op-amp circuit shown in the figure below. What is the output voltage (𝑉0 ) in millivolts if
𝑉1 = 2.5𝑉 and 𝑉2 = 2.25V?

Solution: Ans (5250)

The given circuit is an instrumentation amplifier, the output voltage can be shown as

𝑉𝑜 2𝑅 2𝑅
=1+ ⇒ 𝑉𝑜 = 1 + 𝑉1 − 𝑉2
𝑉1 − 𝑉2 𝑅𝑃 𝑅𝑃
2 × 5000
= 1+ 2.5 − 2.25 = 21 × 0.25 = 5.25 𝑉 = 5250 𝑚𝑉
500

5. A small insect of mass 𝑚 is sitting on the rim of a uniform circular horizontal disk of radius 𝑅 and mass
𝑀. The system is rotating at a constant angular velocity 𝜔𝑖 about a frictionless vertical axis passing
through the centre of the disk. The insect started to crawl towards the centre of the disk. Assume
𝑀/𝑚 = 10, and let the final angular velocity of the system, when the insect reaches the centre of the
disk, be 𝜔𝑓 . What is the value of 100 𝜔𝑓 /𝜔𝑖 ?

Solution: Ans (120)

Initial angular momentum 𝐿𝑖 is given by,

𝑀𝑅 2
𝐿𝑖 = 𝐼𝑖 𝜔𝑖 = + 𝑚𝑅 2 𝜔𝑖
2

𝑀𝑅 2 2
𝑀𝑅 2
𝐿𝑓 = 𝐼𝑓 𝜔𝑓 = + 𝑚0 𝜔𝑓 = 𝜔𝑓
2 2

From conservation of angular momentum,

𝑀𝑅 2 𝑀𝑅 2
𝐿𝑖 = 𝐿𝑓 ⇒ + 𝑚𝑅 2 𝜔𝑖 = 𝜔𝑓
2 2
𝑀𝑅 2
𝜔𝑓 + 𝑚𝑅 2 2 𝑀𝑅 2
2
⇒ = = + 𝑚𝑅 2
𝜔𝑖 𝑀𝑅 2 𝑀𝑅 2 2
2

𝜔𝑓 2𝑚 20
⇒ × 100 = 1 + × 100 = 1 + × 100 = 120
𝜔𝑖 𝑀 100

6. A two-state quantum system has energy eigenvalues ±𝜖 corresponding to normalised states 𝜓 ±. At


time 𝑡 = 0 the system is in quantum state 𝜓+ + 𝜓− / 2. Find the 10000 × probability that the system
will be in the same state at time 𝑡 = ℎ/(6𝜖),where ℎ is the Planck’s constant.

Solution: Ans (5000)

1
𝜓 0 = 𝜓+ + 𝜓−
2

1
𝜓 𝑡 = 𝜓+𝑒 −𝑖∈𝑡/ℏ + 𝜓_ 𝑒 𝑖∈𝑡/ℏ
2

ℎ 𝜖𝑡 1 𝜖𝑡 2𝜋 𝜋
𝑡= ⇒ = ⇒ = =
6𝜖 𝑕 6 ℏ 6 3
1 −𝑖𝜋 𝑖𝜋
𝜓 𝑡 = 𝜓+ 𝑒 3 + 𝜓_ 𝑒 3
2

The probability that the system will be in the same state 𝜓 0 is given by 𝑃 = 𝜓 ∗ 𝑡 𝜓 0

1 −𝑖𝜋 𝑖𝜋 1
= 𝜓+ 𝑒 3 + 𝜓_ 𝑒 3 𝜓+ + 𝜓−
2 2

1 ∗𝑒 −𝑖𝜋 𝑖𝜋 1
= 𝜓+ 3 + 𝜓_∗ 𝑒 3 𝜓+ + 𝜓−
2 2

1 ∗ −𝑖𝜋 𝑖𝜋
= 𝜓 𝑒 3 + 𝜓_∗ 𝑒 3 𝜓+ + 𝜓−
2 +
1 ∗ −𝑖𝜋 𝑖𝜋
= 𝜓 𝜓 𝑒 3 + 𝜓_∗ 𝜓−𝑒 3
2 + +

1 −𝑖𝜋 𝑖𝜋 1 𝜋 𝜋 𝜋 𝜋 𝜋 1
= 𝑒 3 +𝑒 3 = cos − 𝑖 sin + cos + 𝑖 sin = cos =
2 2 3 3 3 3 3 2

⇒10000 × probability that the system will be in the same state at time
1
= 10000 × = 5000
2

7. Some bacteria are added to a bucket at time 10 am. The number of bacteria doubles every minute and
reaches a number 16 × 1015 at 10:18 am. How many seconds after 10 am were there 25 × 1013
bacteria?

Solution: Ans (720)

Let the initial number of bacteria at 𝑡 = 0 is 𝑁𝑜

After each minute the number doubles. So, the number will evolve as: 𝑁𝑜 , 2𝑁𝑜 , 4𝑁𝑜 , 8𝑁𝑜 , 16𝑁𝑜 , ⋯

i.e, 𝑁𝑜 , 2𝑁𝑜 , 22 𝑁𝑜 , 23 𝑁𝑜 , 24 𝑁𝑜 , ⋯

Hence number after 𝑡 minutes will be 𝑁 = 𝑁𝑜 2𝑡

⇒ 16 × 1015 = 𝑁𝑜 × 218

16 × 1015 1015
⇒ 𝑁𝑜 = = 14
218 2

1015
⇒ 25 × 1013 = × 2𝑡
214

⇒ 25 × 214 = 102 × 2𝑡 ⇒ 214 = 4 × 2𝑡 ⇒ 212 = 2𝑡 ⇒ 𝑡 = 12 𝑚𝑖𝑛 = 12 × 60 𝑆𝑒𝑐 = 720 𝑆𝑒𝑐

8. A particle is moving on a one-dimensional discrete lattice with lattice spacing unity. It can move from a
site to its nearest neighbour site every 1/5 second with 𝑝 being the probability to move right and
𝑞 = (1 − 𝑝) being the probability to move left. Consider that the particle starts at origin, 𝑥 = 0 at time
𝑡 = 0. Taking 𝑝 = 3/4, calculate the variance (𝑥 − 𝑥 )2 at time 𝑡 = 5 × 104 seconds, where 𝑥 is
the average position.

Solution: Ans (46875)

The following problem is of random walk in a line. We need to find the probability distribution 𝑝(𝑚, 𝑁)
that the walker will be at position 𝑚 after 𝑁 steps?

Let, 𝑛1 = Number of steps taken right and 𝑛2 = Number of steps taken left

𝑁 = 𝑛1 + 𝑛2

For 𝑚 < 𝑁 there are many ways to start at 0 and go through 𝑁 jumps to nearest neighbor sites and end
up at 𝑚. The possibilities of all the paths taken are independent with each other we have sum all these
probabilities. For all these ways we know that the walker must have made 𝑛1 = 𝑚 + 𝑛2 jumps to the
right and 𝑛2 jumps to the left, and since 𝑛1 + 𝑛2 = 𝑁, the walker must have made

𝑝 𝑛 1 𝑞𝑛 2 = 𝑝 𝑛 1 (1 − 𝑝)𝑛 2

Now, 𝑚 = 𝑛1 − 𝑛2 therefore,

𝑁+𝑚
𝑁 + 𝑚 = 2𝑛1 ⇒ 𝑛1 = jumps to the right
2
𝑁−𝑚
𝑁 − 𝑚 = 2𝑛2 ⇒ 𝑛2 = jumps to the left
2

The probability for a sequence of left and right jumps is the product of the probabilities of the individual
jumps.
𝑁+𝑚 𝑁−𝑚
𝑝𝑛 1 𝑞𝑛 2 = 𝑝 2 𝑞 2

The total number of distinguishable ways to have 𝑛1 steps to the right and 𝑛2 to the left therefore
becomes

𝑁! 𝑁!
=
𝑛1 ! 𝑛2 ! 𝑛1 ! (𝑁 − 𝑛1 )!

The probability of being at position 𝑚 after 𝑁 jumps is therefore given as

𝑁!
𝑃 𝑚, 𝑁 = 𝑝𝑛 1 𝑞𝑛2
𝑛1 ! 𝑛2 !

𝑁! 𝑁+𝑚 𝑁−𝑚
𝑃 𝑚, 𝑁 = 𝑝 2 𝑞 2
𝑁+𝑚 𝑁−𝑚
! !
2 2

which is the so called binomial distribution.

If we know the probability distribution 𝑃 𝑚, 𝑁 , we can calculate all the moments jof m at any fixed
time 𝑁,

From the number of steps to the right


𝑁+𝑚
𝑛=
2
𝑁!
𝑃 𝑚, 𝑁 = 𝑃𝑁 𝑚 = 𝑝𝑛 𝑞 𝑁−𝑛 = 𝑁1 𝐶𝑛 𝑝𝑛 𝑞 𝑁−𝑛
𝑛 ! 𝑁−𝑛 !

and calculate the various moments of 𝑃𝑁 𝑛 . Noting that,

𝑁
𝑁
(𝑝𝑢 + 𝑞)𝑁 = 𝑛 𝑁−𝑛
1 𝐶𝑛 𝑝 𝑞
𝑛 =0
we see that 𝑃𝑁 𝑛 is the coefficient of 𝑢𝑛 in this binomial expansion. Then

⇒ 𝑃𝑁 𝑛 = (𝑝𝑢 + 𝑞)𝑁 𝑢 =1 = 1
𝑛 =0

which shows that 𝑃𝑁 𝑛 is properly normalized to one. The first moment or expectation value of 𝑛 is:

𝑁 𝑁

⇒𝐸 𝑛 = 𝑛 = 𝑛𝑃𝑁 𝑛 = 𝑛 𝑁1𝐶𝑛 𝑢𝑛 𝑝 𝑛 𝑞 𝑁−𝑛 𝑢 =1


𝑛 =0 𝑛 =0

𝑁 𝑁
𝑁
𝑑 𝑑 𝑁
= 1 𝐶𝑛 𝑢 𝑢𝑛 𝑝 𝑛 𝑞𝑁−𝑛 = 𝑢 1 𝐶𝑛 𝑢𝑛 𝑝 𝑛 𝑞 𝑁−𝑛
𝑑𝑢 𝑢 =1 𝑑𝑢
𝑛 =0 𝑛 =0 𝑢 =1

𝑑
= 𝑢 (𝑝𝑢 + 𝑞)𝑁 = 𝑢𝑁(𝑝𝑢 + 𝑞)𝑁−1 𝑝 𝑢=1 = 𝑁(𝑝 + 𝑞)𝑁−1 𝑝 = 𝑁(1)𝑁−1 𝑝 = 𝑁𝑝
𝑑𝑢 𝑢=1

This leads to 𝐸 𝑛 = 𝑛 = 𝑁𝑝
In the same manner we can derive the second moment:

2
𝑑
𝐸 𝑛2 = 𝑛2 = 𝑢 (𝑝𝑢 + 𝑞)𝑁 = 𝑁𝑝 + 𝑁 𝑁 − 1 𝑝 2
𝑑𝑢 𝑢 =1

The variance of the variable 𝑛 is defined by,

𝜍2 = 𝑛− 𝑛 2 = 𝑛2 − 𝑛 2 = 𝑁𝑝 + 𝑁 𝑁 − 1 𝑝 2 − 𝑁 2 𝑝 2

= 𝑁𝑝 + 𝑁 2 𝑝 2 − 𝑁𝑝 2 − 𝑁 2 𝑝 2 = 𝑁𝑝 1 − 𝑝 = 𝑁𝑝𝑞

2
3 1
𝑥− 𝑥 = 𝑁𝑝𝑞 = 5 × 5 × 104 × × = 46875
4 4

9. Two compartments in a cylinder with uniform cross section and total length 102 cm are separated by a
sliding partition which can move but does not allow heat to pass across it. No molecules are present in
either of the compartments. radiation inside each compartment is in thermal equilibrium with its walls.
The walls at the two ends of the cylinder are maintained at temperatures 2000k and 4000k, respectively.
The sides are perfectly insulated. Find the location of the partition, measured from the left end of the
container.
Solution: Ans (96 𝑐𝑚)

The walls radiate photons into the container. Some photons scatter off the walls, with some being
absorbed and new ones being emitted continually, the photons are not conserved. The pressure of the
photon gas doesn’t depend on the volume. And is given by,

𝑃 = 𝐴𝑇 4

Where A is a constant which can be evaluated with the help of quantum statistical mechanics.
Unlike the ideal gas, for which there are three independent variables N, T and V, the photon gas has just
two independent controllable variables T & V.

The number of photons is given by,

𝑁 𝑇, 𝑉 = 𝑟𝑉𝑇 3
Where 𝑟 is a constant,

Therefore, 𝑁1 = 𝑟𝑉1 𝑇13 & 𝑁2 = 𝑟𝑉2 𝑇23

𝑁1 𝑉1 𝑁2 𝑉2
⇒ = 𝑟 𝑇13 & = 𝑟 𝑇23
𝐴 𝐴 𝐴 𝐴
Where, 𝐴 =Area of cross section.

𝑁1 𝑁2
⇒ = 𝑟𝑥𝑇13 & = 𝑟(𝑙 − 𝑥)𝑇23
𝐴 𝐴

From Wein’s displacement law,

𝑏 𝑐 𝑏 𝑐
𝑏 = 𝜆𝑇 ⇒ 𝜆 = ⇒ = ⇒𝜈= 𝑇
𝑇 𝜈 𝑇 𝑏

From Plank’s law, Energy of a photon is

𝑐 𝑕𝑐
𝐸 = 𝑕𝜈 = 𝑕 𝑇 = 𝑇
𝑏 𝑏

The momentum of an electromagnetic wave is given by,

𝐸 𝑕
𝑝= = 𝑇
𝑐 𝑏

The photon striking the partition wall will be absorbed and another photon will be emitted. On an
average over time for a state of thermodynamic equilibrium the number of photons being absorbed will
be equal to the number of photons being emitted as there is no transfer of heat through the walls and the
movable partition. Hence the change in momentum of a photon can be written as,

𝑕𝑇
Δ𝑝 = −𝑝 − 𝑝 = −2𝑝 = −2
𝑏

The photons will move with the speed of light c. therefore, the time taken by the photon to collide with
the partition Δ𝑡 will be same for both sides of the partition wall. Hence the force acting on the partition
wall per photon will be,

Δ𝑝 2𝑕𝑇
Δ𝐹 = − =
Δ𝑡 𝑏Δ𝑡

Hence the total force acting on the left side of partition wall
will be,
2𝑕𝑇1 𝑁1 2𝑕𝑇1
𝐹1 = × = × 𝑟𝑥𝑇13
𝑏Δ𝑡 𝐴 𝑏Δ𝑡

and the total force acting on the right side of partition wall will be,

2𝑕𝑇2 𝑁2 2𝑕𝑇2
𝐹2 = × = × 𝑟(𝑙 − 𝑥)𝑇23
𝑏Δ𝑡 𝐴 𝑏Δ𝑡

For equilibrium, 𝐹1 = 𝐹2
2𝑕𝑇1 2𝑕𝑇2
∴ × 𝑟𝑥𝑇13 = × 𝑟(𝑙 − 𝑥)𝑇23
𝑏Δ𝑡 𝑏Δ𝑡

⇒ 𝑥𝑇14 = 𝑙 − 𝑥 𝑇24 ⇒ 𝑥 2000 4 = 102 − 𝑥 4000 4

4
4000
𝑥 = 102 − 𝑥
2000

⇒ 𝑥 = 16 102 − 𝑥 ⇒ 17𝑥 = 16 × 102 ⇒ 𝑥 = 96 𝑐𝑚

10. A thin film of water having refractive index 𝑛 = 1.333 floats on the surface of a beaker of silicone oil
having refractive index 𝑛𝑠 = 1.40. The arrangement is illuminated by 600 nm light incident normally
from top and a large region of the film appears bright red. What is the minimum possible thickness of the
film(in nm)?
Solution: Ans (214.28 )

If the light is incident at a given angle 𝜃1 ,

𝑑
𝐴𝐵 = 𝐵𝐶 =
cos 𝜃2

The optical path difference (OPD) is given by,

OPD = 𝑛2 𝐴𝐵 + 𝐵𝐶 − 𝑛1 𝐴𝐷

𝐴𝐶
2 = tan 𝜃 ⇒ 𝐴𝐶 = 2𝑑 tan 𝜃
2 2
𝑑

⇒ 𝐴𝐷 = 𝐴𝐶 sin 𝜃1 = 2𝑑 tan 𝜃2 sin 𝜃1

2𝑑
∴ OPD = 𝑛2 − 𝑛1 2𝑑 tan 𝜃2 sin 𝜃1
cos 𝜃2

From Snell’s law,

𝑛1 sin 𝜃1 = 𝑛2 sin 𝜃2

2𝑑
⇒ OPD = 𝑛2 − 𝑛2 2𝑑 tan 𝜃2 sin 𝜃2
cos 𝜃2

2𝑑 sin2 𝜃2 1 − sin2 𝜃2
= 𝑛2 − 𝑛2 2𝑑 = 2𝑛2 𝑑 = 2𝑛2 𝑑 cos 𝜃2
cos 𝜃2 cos 𝜃2 cos 𝜃2

Interference will be constructive if the optical path difference is equal to an integeral multiple of the
wavelength of light, i.e,

𝑚𝜆
2𝑛2 𝑑 cos 𝜃2 = 𝑚𝜆 ⇒ 𝑑 =
2𝑛2 cos 𝜃2

The light is incident normally to the surface therefore, 𝜃1 = 𝜃2 = 0

𝑚𝜆 600 × 𝑚
∴𝑑= = 𝑛𝑚
2𝑛2 2 × 1.40

600 × 1
∴ 𝑑𝑚𝑖𝑛 = 𝑛𝑚 = 214.28 𝑛𝑚
2 × 1.40

You might also like